zz_torna2

New Member
ارسال ها
300
لایک ها
254
امتیاز
0
پاسخ : ماراتن جبر (سطح پیشرفته)

به توان سه برسونید، بعد جملات خوب رو بیارید اینور به توان دو برسونید، بعد am-gm بزنید
am-gm ؟؟؟؟
 

Aref

New Member
ارسال ها
1,262
لایک ها
1,008
امتیاز
0
پاسخ : ماراتن جبر (سطح پیشرفته)

چندتا am-gm
!
:4:
 

mehran88

New Member
ارسال ها
640
لایک ها
1,232
امتیاز
0

zz_torna2

New Member
ارسال ها
300
لایک ها
254
امتیاز
0
پاسخ : ماراتن جبر (سطح پیشرفته)


سوال بعدی.

تمام زوج اعداد طبیعی m,n را بیابید که برای آنها چندجمله ای
بر چندجمله
بخشپذیر باشد.
 

zz_torna2

New Member
ارسال ها
300
لایک ها
254
امتیاز
0
پاسخ : ماراتن جبر (سطح پیشرفته)

سوال بعدی.
آیا چندجمله ایهای درجه
2 , با ضرایب صحیح موجودند به نحویکه هر دوی این چندجملهای ها دارای ریشه صحیح باشند؟
 

zz_torna2

New Member
ارسال ها
300
لایک ها
254
امتیاز
0
پاسخ : ماراتن جبر (سطح پیشرفته)

تمام توابعی رو بیابید که :
 

zz_torna2

New Member
ارسال ها
300
لایک ها
254
امتیاز
0
پاسخ : ماراتن جبر (سطح پیشرفته)








حالا اگه n رو به بینهایت میل بدیم نتیجه میشه

یعنی تابع ثابته .
 

yasamin.m

New Member
ارسال ها
43
لایک ها
4
امتیاز
0
پاسخ : ماراتن جبر (سطح پیشرفته)

همه تابع ها مانند
را طوری پیدا کنید که
 

MBGO

New Member
ارسال ها
247
لایک ها
104
امتیاز
0
پاسخ : ماراتن جبر (سطح پیشرفته)

همه تابع ها مانند
را طوری پیدا کنید که
تابع ما پوشا و یک به یک است.









وقتی این رو توی رابطه ی اصلی مساله بذارید صدق میکنه.
 

MBGO

New Member
ارسال ها
247
لایک ها
104
امتیاز
0
پاسخ : ماراتن جبر (سطح پیشرفته)

الف )مساله ی قبل رو به ازای
حل کنید :

همه تابع ها مانند
را طوری پیدا کنید که
ب) تمام توابع
رو بیابید که در رابطه ی زیر صدق کند :




لطفا لینک ندهید.
 

MBGO

New Member
ارسال ها
247
لایک ها
104
امتیاز
0
پاسخ : ماراتن جبر (سطح پیشرفته)

الف )مساله ی قبل رو به ازای
حل کنید :



ب) تمام توابع
رو بیابید که در رابطه ی زیر صدق کند :




لطفا لینک ندهید.

خب من لینک جواب این سوال زو بعدا ت همین پست میذارم. به نظرم بیخیال این سوال و از این به بعد چندجمله ای سوال بذاریم.پس دیگه سوال نامساوی یا دستگاه معادلات یا تابع نذارید. برای جواب ها هم یا جواب رو کامل بذارید...یا کلیاتی ازش رو بذارید یا لینکش.اگر هم لینک میذارید ایده ی کلی اینجور سوال ها رو بگید.

ثابت کنید چندجمله ای روبه رو تحویل ناپذیر است:
 

math

New Member
ارسال ها
1,129
لایک ها
1,096
امتیاز
0
پاسخ : ماراتن جبر (سطح پیشرفته)

خب من لینک جواب این سوال زو بعدا ت همین پست میذارم. به نظرم بیخیال این سوال و از این به بعد چندجمله ای سوال بذاریم.پس دیگه سوال نامساوی یا دستگاه معادلات یا تابع نذارید. برای جواب ها هم یا جواب رو کامل بذارید...یا کلیاتی ازش رو بذارید یا لینکش.اگر هم لینک میذارید ایده ی کلی اینجور سوال ها رو بگید.

ثابت کنید چندجمله ای روبه رو تحویل ناپذیر است:

برهان خلف یزنید :


پس بدون کم شدن از کلیت مساله میتوان فرض کرد
حال اگر داشته باشیم
ثابت کنید
پس تناقض ....:D
 

AHZolfaghari

Well-Known Member
ارسال ها
935
لایک ها
1,654
امتیاز
93
پاسخ : ماراتن جبر (سطح پیشرفته)

باز هم شاهد آن شدم که یک تایپیک با ارزش به امان خدا رها شد !!!
از راهبران و مشاورین محترم درخواست میکنم نگذارند رها بشه.
 

MiMi1376

New Member
ارسال ها
81
لایک ها
48
امتیاز
0
پاسخ : ماراتن جبر (سطح پیشرفته)

همه توابع
را بیابید که برای هر
و
و
و
بزرگتر از صفر که در
صدق می کنند داریم
(خیلی سخت نیست)
 

MiMi1376

New Member
ارسال ها
81
لایک ها
48
امتیاز
0
پاسخ : ماراتن جبر (سطح پیشرفته)

چرا هیج کی اینو حل نمی کنه ؟؟

-
 

MiMi1376

New Member
ارسال ها
81
لایک ها
48
امتیاز
0
پاسخ : ماراتن جبر (سطح پیشرفته)

فک کنم این ماراتن طلسم شده.
 

AHZolfaghari

Well-Known Member
ارسال ها
935
لایک ها
1,654
امتیاز
93
پاسخ : ماراتن جبر (سطح پیشرفته)

همه توابع
را بیابید که برای هر
و
و
و
بزرگتر از صفر که در
صدق می کنند داریم
(خیلی سخت نیست)
من به این شکل حل کردم اینو :

اول اثبات کردم که

به راحتی میشه اینو بدست اورد.بعد (f(1 رو هم به راحتی بدست میاریم که میشه یک.
بعد به جای y می ذاریم 1 و به جای x می ذاریم x^2 و به جای w,z هم میذاریم x
حالا یه معادله بر حسب (x , f(x بدست میاد که دو تا نتیجه میده . یا
یا

حالا میخوایم اثبات کنیم f همه اعداد به یکی از این فرم هاست پس میایم میگیم مثلا یه a وجود داره که
و یه b وجود داره که

حالا میایم به جای w,z میذاریم
و به جای x میذاریم a و بجای y میذاریم b.
حالا یه معادله بدست میاد که فقط
توش مجهوله که باتوجه به رابطه ای که داشتیم یا میشه ab یا معکوسش که الان هیچیک تو معادله ما صدق نمی کنه پس تناقض میده پس فرم همه ی اعداد به یک شکل هستش پس دو تا جواب داره:
و


سوالش هم سوال چهار المپیاد جهانی 2008 بوده.
 
آخرین ویرایش توسط مدیر

AHZolfaghari

Well-Known Member
ارسال ها
935
لایک ها
1,654
امتیاز
93
پاسخ : ماراتن جبر (سطح پیشرفته)

از دوستان خواهش میکنم اینجا سوال بذارن تا حل کنیم با هم دیگه . این تاپیک با ارزش رو ولش نکنید

---- دو نوشته به هم متصل شده است ----

از دوستان خواهش میکنم اینجا سوال بذارن تا حل کنیم با هم دیگه . این تاپیک با ارزش رو ولش نکنید

---- دو نوشته به هم متصل شده است ----

از دوستان خواهش میکنم اینجا سوال بذارن تا حل کنیم با هم دیگه . این تاپیک با ارزش رو ولش نکنید

---- دو نوشته به هم متصل شده است ----

از دوستان خواهش میکنم اینجا سوال بذارن تا حل کنیم با هم دیگه . این تاپیک با ارزش رو ولش نکنید

---- دو نوشته به هم متصل شده است ----

از دوستان خواهش میکنم اینجا سوال بذارن تا حل کنیم با هم دیگه . این تاپیک با ارزش رو ولش نکنید

---- دو نوشته به هم متصل شده است ----

از دوستان خواهش میکنم اینجا سوال بذارن تا حل کنیم با هم دیگه . این تاپیک با ارزش رو ولش نکنید

---- دو نوشته به هم متصل شده است ----

از دوستان خواهش میکنم اینجا سوال بذارن تا حل کنیم با هم دیگه . این تاپیک با ارزش رو ولش نکنید
 

ash1374

New Member
ارسال ها
253
لایک ها
422
امتیاز
0
پاسخ : ماراتن جبر (سطح پیشرفته)

x,y,z حقیقی و مثبتند. ثابت کنید :

اگر استقبال بشه ان شا الله باز هم سعی می کنم سوال بذارم.
 
بالا